Answer to Question #110948 in Electric Circuits for James

Question #110948
Calculate (a) the value of the resistor Rx, such that the total power dissipated in the circuit is 2.5KW, and (b) the current flowing in each of the four resistors . R1=15ohms, R2=10ohms, R3=38ohms and R4=Rx.
1
Expert's answer
2020-04-21T10:56:36-0400

As per the given question,

Here value of V and the diagram was not given, so I assumed that the given diagram is like given below,


"R_1=15\\Omega, R_2=10\\Omega, R_3=38\\Omega R_4=R_x."

a) P=VI

"I=\\dfrac{2.5\\times 1000}{250}=10A"

ii) "250=10(\\dfrac{15\\times 10}{15+10})+10\\times\\dfrac{38\\times R_x}{38+R_x}"

"\\Rightarrow 25=\\dfrac{150}{25}+\\dfrac{38R_x}{38+R_x}"

"\\Rightarrow 25=6+\\dfrac{38R_x}{38+R_x}"

"\\dfrac{38R_x}{38+R_x}=19"

"\\Rightarrow 38R_x=38\\times 19+R_x\\times 19"

"\\Rightarrow 19R_x=38\\times 19"

"\\Rightarrow R_x=38\\Omega"


Need a fast expert's response?

Submit order

and get a quick answer at the best price

for any assignment or question with DETAILED EXPLANATIONS!

Comments

No comments. Be the first!

Leave a comment

LATEST TUTORIALS
New on Blog
APPROVED BY CLIENTS